y=x^{2} +2x-8 determine the axis of symmetry, the vertex point and find the roots by factoring

Answers

Answer 1

Answer:

We have axis of symmetry in [tex]x=-1[/tex]

The vertex point is [tex](-1, -9)[/tex]

The roots by factoring are

[tex]x=-4[/tex]

[tex]x=2[/tex]

Step-by-step explanation:

We have the function:

[tex]f(x)=x^2+2x-8[/tex]

Factoring the quadratic equation:

[tex]y=(x+4)(x-2)[/tex]

When [tex]y=0[/tex]

[tex]0=(x+4)(x-2)[/tex]

[tex]x=-4[/tex]

[tex]x=2[/tex]

The vertex point is [tex](h, k)[/tex]

From the equation, we have [tex]a=1,\:b=2 \text{ and }\:c=-8[/tex]

[tex]$h=-\frac{b}{2a}$[/tex]

[tex]$h=-\frac{2}{2\cdot \:1}$[/tex]

[tex]h=-1[/tex]

We also got the axis of symmetry

In order to find [tex]k[/tex] just use the [tex]$h=x_{vetex}$[/tex]:

[tex]k=(-1)^2+2(-1)-8\\k=1-2-8\\k=-9[/tex]

Once [tex]a>0[/tex], we have the minimum at [tex](-1, -9)[/tex]

Answer 2

Answer:

x = -1  axis of symmetry

(-1,-9) vertex

x = -4           x=2   are the roots

Step-by-step explanation:

y = x^2 + 2x - 8

The axis of symmetry is

h = -b/2a

h = -2/ (2*1) = -2/2 =-1

x = -1

The x coordinate of the vertex is at the axis of symmetry

To find the y value substitute into the function

y = (-1)^2 +2(-1) -8

y = 1-2-8

y = -9

The vertex is at ( -1,-9)

Factor

What 2 number multiply to -8 and add to 2

4*-2 = -8

4+-2 = 2

y = ( x+4) (x-2)

Using the zero product property to find the roots

0 = ( x+4) (x-2)

x+4 =0    x-2 =0

x = -4           x=2   are the roots


Related Questions

Alice invests $15,000 at age 30 from the signing bonus of her new job. She hopes the investment will be worth $30,000 when she turns 40. If the interest compounds continuously, approximately what rate of growth will she need to achieve her goal? Round to the nearest tenth of a percent.

Answers

Answer:

6.9

Step-by-step explanation:

A movie ticket sold for $6.50 ten years ago.
Now a movie ticket costs $9.50. What is the
percent increase?

Answers

Answer: 46%

Step-by-step explanation:

Hi, to answer this question we have to write an equation:

The price of a movie ticket ten years ago (6.50) multiplied by a percentage in decimal form (x) is equal to 9.50.

6.50x = 9.50

Solving for x  

x =9.50/6.50

x = 1.46

Multiplying by 100 to obtain the percent form:

1.46 x 100 = 146%

Finally we have to subtract 100 to obtain the percent increase:

146-100 = 46%

Feel free to ask for more if needed or if you did not understand something.

QUICK ANSWER PLEASE The left and right page numbers of an open book are two consecutive integers whose sum is 269. Find these page numbers.

Answers

Answer:

134 and 135

Because 269/2 is 134.5, so you know that one is 134 and the other is 135.

The page numbers are 134 and 135

What are integers?

Any positive or negative number without fractions or decimal places is known as an integer, often known as a "round number" or "whole number."

Given:

The left and right page numbers of an open book are two consecutive integers whose sum is 269.

Let two consecutive integers are n, n+1.

Then,

the sum of two consecutive integers is 269.

According to the question,

n + n + 1 = 269

2n + 1 = 269

2n = 268

n = 134

The integers are 134 and 135.

Therefore,  integers are 134 and 135.

To learn more about the integers;

brainly.com/question/1768254

#SPJ2

Click an item in the list or group of pictures at the bottom of the problem and, holding the button down, drag it into the
correct position in the answer box. Release your mouse button when the item is place. If you change your mind, drag
the item to the trashcan. Click the trashcan to clear all your answers.
Enter the equation of the circle described below.
Center (0, 0), radius = 4

Answers

The standard equation of a unit circle is

x²+y²=1

So, if we want a radius of 4, we need to square it.

so we have x²+y²=16

Answer:

x^2 + y^2 = 16

Step-by-step explanation:

The equation of a circle is given by

(x-h)^2 + (y-k)^2 = r^2  where (h,k) is the center and r is the radius

(x-0)^2 + (y-0)^2 = 4^2

x^2 + y^2 = 16

Solve the following system:
y=x + 3
2x + y = 9

Answers:
(2,5)
(5,2)
(-2,5)
(2,-5)

Answers

Answer

your anwser would be (2,5)

Step-by-step explanation:

y - x = 3 ...1

2x + y = 9 ...2

...2 - ...1

2x + y -y + x = 9 - 3

3x = 6

x = 2

replace x you will receive y value

Two bottles of water and three small bags of fruit cost a total of £2.55. If a bottle of water costs 15p more than a bag of fruit, what is the cost of a bag of fruit?

Answers

Answer:

The cost of a bag of fruit is 45p

Step-by-step explanation:

Let w be the number of water bottles.

Let f be the number of bags of fruits.

"Two bottles of water and three small bags of fruit cost a total of £2.55"

2w + 3f = 2.55

"a bottle of water costs 15p more than a bag of fruit"

w = 0.15 + f

Substitute the second formula to the first and solve.

2w + 3f = 2.55

2(0.15 + f) + 3f = 2.55

0.3 + 2f + 3f = 2.55

0.3 + 5f = 2.55

5f = 2.55 - 0.30

5f = 2.25

f = 2.25 ÷ 5

f = 0.45

Substitute to the second formula again and solve for w.

w = 0.15 + 0.45

w = 0.6

Now verify:

2w + 3f = 2.55

2(0.6) + 3(0.45) = 2.55

1.2 + 1.35 = 2.55

2.55 = 2.55 (Correct)

what is factored form of the expression 16st+15t

Answers

Answer:

t(16s+15)

Step-by-step explanation:

The highest common factor of 16st+15t is t.

t(16s) + t(15)

Factor.

t(16s+15)

Answer:

t ( 16s+15)

Step-by-step explanation:

=> 16st + 15t

Taking t common

=> t ( 16s+15)

3. If you’re good at geometry could you please help me?

Answers

Answer:

Step-by-step explanation:

[tex]a^2+b^2=c^2\\14^2+x^2=c^2\\14^2=196\\39^2=1521\\x^2=1521-196\\=1325\\\sqrt{1325} \\x=36.4ft[/tex]

PLEASE HELP ME THIS IS URGENT I AM BEING TIMED!

Brandon entered a contest for free math materials. A total of 758 people each put in one entry, and there will be only one winner. What is the probability of Brandon winning the contest? Write your answer as a fraction. Represent a fraction using the / symbol.
Example: 1/2

Answers

Answer:

1/758

Step-by-step explanation:

Part/whole

Answer:

1/758

Step-by-step explanation:

hope that helps

After seeing this video, another dog owner trained his dog, Lightning, to try to break Tillman’s skateboarding record.
Lightning’s fastest recorded time was on a 75-meter stretch where it took him 15.5 seconds. Based on these data,
did Lightning break Tillman’s record for fastest dog on a skateboard? Explain how you know.

Answers

Answer:

75/15.5 = 4.83870967742 =4.84 meters per second.

This is a faster time than 5 meters per second.

Step-by-step explanation:

Please answer correctly !!!! Will mark brainliest !!!!!!!

Answers

Answer:

Think it's 7.

Step-by-step explanation:

Because 5-4=1 so it have to be 7.

7+1=8

i need help with this PLZ!!!!!!!​

Answers

Answer:

3. 91.125

4. 30.375

Step-by-step explanation:

just multiply all of the sides

MARK ME BRAINLIEST

Answer:

Given below

Step-by-step explanation:

Area of cube = s*s*s

= 4.5x4.5x4.5=

91.125 mm^3

Area of cuboid is lbh

= 6*2.25*2.25

= 30.375 mm^3

Margo has 15 pounds of clay, Tim has 10 pounds of clay, Tray has 13 pounds of clay, Lila has 14 pounds of clay, and Jerry has 13 pounds of clay. How can they redistribute the clay so that they all have the same amount? Check all that apply.

Answers

Answer:

13 clay for each person

Step-by-step explanation:

Add all the clay to find the total.

15+10+13+14+13= 65 clay in total

There are 5 people in total.

Divide 65 by 5 to find the distribution for each person.

65÷5= 13

Each person get 13 clay after redistribution.

They redistribute the clay then  Each person gets 13 clay after redistribution.

We have given that,

Margo has 15 pounds of clay, Tim has 10 pounds of clay, Tray has 13 pounds of clay, Lila has 14 pounds of clay, and Jerry has 13 pounds of clay.

What is the total of the clay?

Add all the clay to find the total.

15+10+13+14+13= 65 clay in total

There are 5 people in total.

Divide 65 by 5 to find the distribution for each person.

[tex]65/5= 13[/tex]

Each person gets 13 clay after redistribution.

To learn more about the distribution visit:

https://brainly.com/question/5512053

#SPJ2

What is the value of y for the point with an x-value of 3?

Answers

Answer:

for x =3  y = 4

Step-by-step explanation:

Go across to x = 3

Then go up until you reach a dot

The got left to read the y value

for x =3  y = 4

Use the diagram to estimate the perimeter, rounding each number to the nearest hundred
5,534 11
1,666 ft
1,566 ft
5,534 ft

Answers

Answer:

14,300 ft

Step-by-step explanation:

Here are the following perimeters rounded to the nearest hundred:

5534 rounds down to 5500

1666 rounds up to 1700

1566 rounds up to 1600

5534 rounds down to 5500

The sum of these is the following:

5500 ft + 1700 ft + 1600 ft + 5500 ft = 14,300 ft

A rectangular prism has a length of 9 cm a width of 5 cm and its volume is 180 cubic cm what is the height of the prism

Answers

Answer:

The height is 4cm

Step-by-step explanation:

Multiply 9 x 5 which is 45

Then divide 180 from 45

You get 4

To check your work, multiply 9 x 5 x 4 and you get 180! hopes this helps

Solve the equation, –(x∕4) = 5, for x. Which of the following properties of equality did you apply? Question 7 options: A) Multiplication B) Multiplication and subtraction C) Division and addition D) Division

Answers

Answer:

A) Multiplication

The solution of given equation  x= -20

Step-by-step explanation:

Step(i):-

Given equation  - ( x /4) = 5

Multiply '-1' on both sides, we get

                     ⇒    - ( (- x /4) ) = - 5

                     ⇒    [tex]\frac{x}{4} = -5[/tex]

Step(ii):-

Multiply '4' on both sides, we get  

                     ⇒    [tex]\frac{x}{4} X 4= -5 X 4[/tex]

Cancellation '4' on left side , we get

                          x = -20

Conclusion:-

The solution of given equation  x= -20

Answer:

A

Step-by-step explanation:

Shiva brought $37.00 to the art supply store. She bought a brush, a sketchbook, and a paint set. The brush was 1 3 as much as the sketchbook, and the sketchbook cost 1 2 the cost of the paint set. Shiva had $2.00 left over after buying these items. What was the cost of each item?

Answers

Answer:

b= 3.5

s= 10.5

p=21

Step-by-step explanation:

b for brush

s for sketch book

p for paint set

b=1/3s

s= 1/2 p

b+s+p=37-2

1/3 s +s+2 s =35

1/3 s +s +2 s =35

10/3 s=35

s= 35*3/10

s= 10.5

b=1/3 s = 10.5/3 = 3.5

s= 1/2 p so p= 2s = 2( 10.5) = 21

10.5+3.5 + 21 = 35

20 cm
8 cm
8 cm
20 cm
15 cm
Find the area of the arrow above.
[? ] square centimeters Irregular figures please help explain answer​

Answers

Answer:

Step-by-step explanation:

Area of the base = l x w

= 20 x 18 = 360 cm^2

Area of the triangle = hb/2

(31 x 20)/2 = 620/2 = 310 cm^2

Area of the arrow = 360 + 310

= 670 cm^2

Answer:

610 squared centimeters

Step-by-step explanation:

21. If the area of the plane shape below is
36cm, find its height.​

Answers

Answer:

4 cm

Step-by-step explanation:

The figure is a parallelogram with area (A) calculated as

A = bh ( b is the base and h the perpendicular height )

Here A = 36 and b = 9 , thus

9h = 36 ( divide both sides by 9 )

h = 4 cm

Answer:

Step-by-step explanation:

considering the breadth of the shape as 5cm and length as 9cm...

now... for finding the height of the plane shape...

we need to use the formula for area of a trapezium...

so. area of a trapezium = 1/2 x height x sum of parallel sides

we are given area of trapezium = 36cm^2

SO SUBSTITUTING THE EQUATION...

36 = 1/2 x height x (9+9)

height = 4cm

1.
Look at the number line shown below.
32
Which word problems or inequalities could have a solution represented by the
number line?
Select all that apply.
A. Christopher has read for more than 32 minutes.
B. Christopher has read for less than 32 minutes.
C. Christopher has read for 32 minutes.
D. x < 32
E. x > 32
F. X = 32

Answers

Answer:

A -> E

B -> D

C -> F

Step-by-step explanation:

In all cases, we represent minutes that Christopher has read with variable x

Christopher has read for more than 32 minutes  corresponds with the inequality: x > 32 ."More than" is equivalent to "greater than", which is symbolized by ">".

Christopher has read for less than 32 minutes, corresponds with the inequality: x < 32 . "Less than" symbolized by "<".

Christopher has read for 32 minutes corresponds with the equality: X = 32. Here the amount of time is fixed, so it agrees with an equality instead of an inequality.

Answer: ABDE

Step-by-step explanation:

Which is a true statement about a 45-45-90 triangle?
O A. Each leg is 3 times as long as the hypotenuse.
O B. Each leg is 12 times as long as the hypotenuse.
O C. The hypotenuse is 3 times as long as either leg.
O D. The hypotenuse is v2 times as long as either leg.

Answers

The answer should be
D. The hypotenuse is v2 times as long as either leg.

The hypotenuse is √2 times as long as either leg of a 45-45-90 triangle

What is a Triangle?

A triangle is a plane figure or polygon with three sides and three angles.

A Triangle has three vertices and the sum of the interior angles add up to 180°

Let the Triangle be ΔABC , such that

∠A + ∠B + ∠C = 180°

The area of the triangle = ( 1/2 ) x Length x Base

For a right angle triangle

From the Pythagoras Theorem , The hypotenuse² = base² + height²

if a² + b² = c² , it is a right triangle

if a² + b² < c² , it is an obtuse triangle

if a² + b² > c² , it is an acute triangle

Given data ,

Let the triangle be represented as ΔABC

Now , the measure of ∠ACB = measure of ∠BAC = 45°

And , the measure of ∠ABC = 90°

So , the triangle is a 45-45-90 triangle

Now , the measure of the sides be AB = BC = 1 unit

And , From the Pythagoras Theorem , The hypotenuse² = base² + height²

if a² + b² = c² , it is a right triangle

The measure of side AC ( hypotenuse ) = √ ( AB )² + ( BC )²

The measure of side AC ( hypotenuse ) = √ ( 1 + 1 )

The measure of side AC ( hypotenuse ) = √2 units

Hence , the measure of the hypotenuse of 45-45-90 triangle is √2 units

To learn more about triangles click :

https://brainly.com/question/16739377

#SPJ7

Need help ASAP PLEASE WILL MARK BRAINLEST What is the value of the expression 3/6^2 +7x4-5

Answers

Answer: 277/12

Step-by-step explanation:

Use the order of operations or pemdas (parenthesis, exponents, multiplication, division, addition, subtraction) to solve this equation:

3 / 6^2 +7 x 4 - 5        

there are no parenthesis, so exponents

= 3 / 36 + 7 x 4 - 5            

now multiplication and division(from left to right)

= 3 / 36 + 7 x 4 - 5

= 1/12 + 28 - 5                  

addition and subtraction are next(it doesn't matter the order in this case):

1/12 + 23

=  1/12 + 276/12

= 277/12

i hope this helps

Ten less the quotient If number and 3 is 6

Answers

Answer:

Quotient of a number and 3 = x/3

Ten less the quotient of a number and 3 will equal 6

So...

10 - x/3 = 6

    + x/3      +x/3

10 = x/3 +6

-6              - 6

3 × 4=(x/3) × 3

12 = x

Answer:

The number is 48

Step-by-step explanation:

The number be x

Quotient of the number and 3 : x/3

Ten less the quotient of the number and x :  x/3 -10

[tex]\frac{x}{3}-10=6\\\\\frac{x}{3}=6+10\\\\\frac{x}{3}=16\\\\x=16*3\\\\x=48[/tex]

If you are given the graph of h(x) = log. x, how could you graph m(x) = log2(x+3)?
O Translate each point of the graph of h(x) 3 units up.
O Translate each point of the graph of h(x) 3 units down.
O Translate each point of the graph of h(x) 3 units right.
O Translate each point of the graph of h(x) 3 units left.

Answers

Answer:

Option (d).

Step-by-step explanation:

Note: The base of log is missing in h(x).

Consider the given functions are

[tex]h(x)=\log_2x[/tex]

[tex]m(x)=\log_2(x+3)[/tex]

The function m(x) can be written as

[tex]m(x)=h(x+3)[/tex]           ...(1)

The translation is defined as

[tex]m(x)=h(x+a)+b[/tex]           .... (2)

Where, a is horizontal shift and b is vertical shift.

If a>0, then the graph shifts a units left and if a<0, then the graph shifts a units right.

If b>0, then the graph shifts b units up and if b<0, then the graph shifts b units down.

On comparing (1) and (2), we get

[tex]a=3,b=0[/tex]

Therefore, we have to translate each point of the graph of h(x) 3 units left to get the graph of m(x).

Hence, option (d) is correct.

What is the value of p in the equation?
4 (8-4p)+20= 12
-20
5
10

Answers

Answer:

2.5

Step-by-step explanation:

I think you're missing a value because none of those work. 2.5 does

4*2.5 = 10

8 - 10 = -2

4 (-2) = -8

-8 + 20 = 12

There is a group of $5$ children, where two of the children are twins. How many ways can I distribute $6$ identical pieces of candy to the children, if the twins must get an equal amount of candy?

Answers

Answer:

50

Step-by-step explanation:

Suppose that the kids get $x_1,x_2,x_3,x_4,x_4$ pieces of candy, respectively (each twin received $x_4$ pieces). Then we seek all quadruples $(x_1,x_2,x_3,x_4)$ satisfying

\[x_1+x_2+x_3+2x_4 = 6.\]We proceed by casework, based on the number of candies the twins receive.

If $x_4 = 0$, then there are 6 candies to distribute to 3 kids. This is like arranging 6 C's and 2 |'s (dividers), so there are $\binom{8}{2} = 28$ possible distributions.

If $x_4 = 1$, there are 4 candies remaining for the other 3 kids, so there are $\binom{6}{2} = 15$ possible distributions.

If $x_4 = 2$, there are 2 candies remaining, so there are $\binom{4}{2} = 6$ possible distributions.

If $x_4 = 3$, there are 0 candies remaining, for 1 possible distribution.

Adding all of these results from the separate cases, there are a total of

\[28 + 15 + 6 + 1 = \boxed{50}\].

What is the slope of a line parallel to the line whose equation is 4x – 2y = 7?

Answers

Answer:

Use the slope-intercept form  

y = m x + b

to find the slope  m  and y-intercept  b .

Slope:  

2

y-intercept:  

− 7 2

Lines with the same slope are parallel.

parallel = 2

Answer:2

First, solve for y

-2y=7-4x

Y=2x-7/2

y=mx+b

What we really want to know is the slope(m)

The equation y = 5 represents the graph of a line perpendicular to the y-axis and passing through the point (1,5).
a) True
b) False

Answers

Answer:

False

Step-by-step explanation:

A perpendicular line parallel to the y- axis has equation

x = c

where c is the value of the x- coordinates the line passes through.

The line passes through (1, 5) with x- coordinate 1, thus

x = 1 ← equation of perpendicular line

Answer:

B. False

Step-by-step explanation:

I hope this helps!

Short version of the answer.

You and some friends have $30. You want to order large
pizzas (p) that are $9 eachand drinks (d) that cost $1 each.
Write and graph an inequality that shows how many pizzas
and drinks can you order.?

Answers

You can buy 3 pizzas and 3 drinks
Other Questions
if you flip three coins, what is the probability that you'll get all three tails Determine the total number of roots of each polynomial function using the factored form.f (x) = (x + 5)^3(x - 9)(x + 1) Adam's interview with the hiring manager is going well. However, he wants to ensure that his skills and work history are memorable.What technique should Adam use to stand out from the competition? Show pictures of his family members and pets Speak loudly to get attention Tell concise stories to showcase his experience Wear an unusual outfit to be noticeable When using a straight ladder, it is recommended that the base of the ladder by placed approximately 1/4 the length of the entire ladder away from the wall. The sentence below has been rewritten several ways.Some of them work in English, and some don't. Only one answer,however,inverts the subject and verb.Choose the answer with the inverted subject-verb pattern.A)walked my father around the block.B)around the block walked my father.C)walked around the block my father.D)around the block my father walked. A can of pumpkin pie mix contains a mean of 30 ounces and a standard deviation of 2 ounces. The contents of the cans are normally distributed.. Supposed four can of pumpkin pie mix are randomly selected What is the probalility that the average content of cans in sample is between 28.0 ounces and 31.5 NEED ANSWER FAST PLSSS (reading like a historian-Korean war) please help asap!!Which of these textbooks do you find more trustworthy? Why? (Use specificexamples from each text to support your answer). Find the missing value A) 14B) 12C) 10D) 13 Suppose a population of 250 crickets doubles in size every 3 months how many crickets will there be after 2 years ? Cullumber Company is considering buying equipment for $220000 with a useful life of 5 years and an estimated salvage value of $6000. If annual expected income is $28000, the denominator in computing the annual rate of return is $226000. $110000. $113000. $220000. The data represents the daily rainfall (in inches) for one month. Construct a frequency distribution beginning with a lower class limit of 0.00 and use a class width of 0.20.Does the frequency distribution appear to be roughly a normaldistribution?data0.3800.220.06000.2100.530.18000.02000.24000.01001.280.2400.190.53000.240Daily Rainfall(in inches)Frequency0.00 dash 0.190.00-0.19nothing0.20 dash 0.390.20- 0.39nothing0.40 dash 0.590.40-0.59nothing0.60 dash 0.790.60-0.79nothingDaily Rainfall(in inches)Frequency0.80 dash 0.990.80-0.99nothing1.00 dash 1.191.00-1.19nothing1.20 dash 1.391.20-1.39nothingDoes the frequency distribution appear to be roughly a normaldistribution?A. No, although the distribution is approximately symmetric, the frequencies do not start low, increase to some maximum frequency, then decrease.B. No, although the frequencies start low, increase to somemaximum, then decrease, the distribution is not symmetric.C. No, the distribution is not symmetric and the frequencies do not start off low.D. Yes, all of the requirements are met. What are the advantages of a closed question? A. Closed questions allow the respondent to go in-depth with their answers. B. It is possible to automate the collection of results for closed questions. C. Closed questions allow for new solutions to be introduced. D. It is easy to quantify and compare the results of surveys with closed questions. Waves can be classified as transverse, longitudinal, or surface waves. This depends on the ___________ of movement of the ________ in the medium compared to the direction of the energy. Does economic globalization benefit all countries equally? Why or why not? Provide some evidence. In a data distribution, the first quartile, the median and the means are 30.8, 48.5 and 42.0 respectively. If the coefficient skewness is - 0.38What is the appropriate value of the third quartile Suppose a (the word 'a' means starting amount is 1) radioactive substance decays at a rate of 5% per hour, what percent of the substance is left after 10 hours? (make sure you convert your answer from decimal to percent and round to the nearest whole percent) These two figures are the image and pre-image of adilation.Find the value of x.4 m6 m8 m9 m Which economic problem did many Latin American nations face in the years following World War II? 137355A) a large middle class B) low agricultural output C) wide gaps between rich and poor What is the sum of Negative 8 + 6?